Political scientist: All governments worthy of respect allow their citizens to dissent from governmental policies. N...

Isha on May 22, 2020

explanation

Can you explain why the answer choice is correct and why the remaining are incorrect

Reply
Create a free account to read and take part in forum discussions.

Already have an account? log in

on May 25, 2020

Hello @ishadoshi,

Absolutely, there are some important sufficient and necessary principles involved here.

Government worthy of respect = GWR
Allows citizen dissent = ACD
Leaves minorities unprotected = LMU

Premise 1: GWR ---> ACD
Premise 2: GWR ---> not LMU

The conclusion introduces a government that protects minorities, or in other words (not LMU). Where do we see this term in our diagram? It is the necessary condition of premise 2. The political scientist thinks that the necessary condition (not LMU) leads to the sufficient condition (GWR), which in turn leads to (ACD).

C: not LMU ---> GWR ---> AD

Necessary does not lead to sufficient! This is a misunderstanding of conditional reasoning, and a very common error on this test that you need to understand. We must look for an answer choice that commits the same error.

A. This is a valid argument. It gives us a sufficient and necessary statement. We are given the sufficient, so the necessary will follow.

B.
JM ---> CI
JM ---> CRM

CRM ---> JM ---> C
Do you see the same error here? The author thinks that CRM leads to JM, which in turn leads to CI. This is the same reversal of sufficient and necessary conditions from above! B is correct.

C.
P: CDC ---> PLM ---> not VPL
C: not CDC ---> VPL

The author thinks that negating the sufficient condition (CDC), forces the negation of the necessary condition (VPL). Negating the sufficient has no effect on the necessary! This is an important error to understand, but it is not the same one from the stimulus.

D.
I --- some --- not SA
I ---> not PA

There is a quantifier here that is not present in the stimulus, so that is my first clue that this is not the right answer. There is an error here, however. The author is trying to make a contrapositive of the quantifier statement, which is not valid. Contrapositives only apply to conditional statements.

PA ---> not I (valid)
not I ---> SA (Strange attempt at a contrapositive that is not valid)

E.
FPN ---> Conceal some character thoughts
TPN --- some --- Reveal all motives

Just because some third person narratives have this characteristic, we can't make an argument that this "should" happen. This is flawed, but not the flaw we are looking for.